PT2.S2.Q13 - If retail stores experience a decrease in revenue

Pretzel LogicPretzel Logic Member
edited June 2021 in Logical Reasoning 227 karma

I couldn't understand why I got this wrong until I started writing this, so I figured I'd publish just in case it helps others.

Decrease in revenues > Prices risen beyond what people can afford > Salaries not kept pace.

Contrapositive: Salaries kept pace > Prices do not rise beyond level people can afford > No decrease in revenue

Question: If salaries have kept pace during last year, what MBT?

(B) Incorrect: Retail stores will not experience a drop in retail sales this holiday season

(C) Correct: Prices in retail stores have not risen beyond the level that most people can afford during this holiday season.

Sign In or Register to comment.